GRE网络课堂学习笔记-逻辑[7]
来源:优易学  2010-1-21 16:52:03   【优易学:中国教育考试门户网】   资料下载   外语书店

第四讲 规则题

1. Questions 12-17

A hobbyist is stocking her aquarium with exactly three fish of different types and with exactly two species of paints. The only fish under consideration are a G , an H , a J , a K and an L and the only kinds of plants under consideration are of the species W , X , Y , and Z . She will observe the following conditions:
If she selects the G, she can select neither the H nor a Y.
She cannot select the H unless she selects the K.
She cannot select the J unless she selects a W.
If she selects the K, she must select an X.

(12). Which one of the following is an acceptable selection of fish and plants for the aquarium?

   Fish    Plants
(A) G, H, K   W, Y
(B) G, J, K   W, X
(C) G, J, L   X, Z
(D) H, J, L   W, Z
(E) H, K, L   Y, Z
(13). If the hobbyist selects the H, which one of the following must also be true?

(A) She selects at least one W.
(B) She selects at least one X.
(C) She selects the J, but no Y's.
(D) She selects the K ,but no X's.
(E) She selects at least one X, but no Y's.

(14). If the hobbyist selects both X's and Z's, which one of the following could be the group of fish she selects?

(A) G, H, K
(B) G, J, K
(C) G, K, L
(D) H, J, L
(E) J, K, L

(15). The hobbyist could select any of the following groups of fish for the aquarium EXCEPT:

(A) G, K, L
(B) H, J, K
(C) H, J, L
(D) H, K, L
(E) J, K, L

(16). If the hobbyist selects a Y, which one of the following must be the group of fish she selects?

(A) G, H, K
(B) H, J, K
(C) H, J, L
(D) H, K, L
(E) J, K, L

(17). The hobbyist could select ant of the following plant combinations EXCEPT:

(A) W and X
(B) W and Y
(C) W and Z
(D) X and Y
(E) X and Z

KEYS: BBCCDB

责任编辑:sealion1986

文章搜索:
 相关文章
热点资讯
热门课程培训
论坛新帖